Flawed Parallel Reasoning Questions - - Question 18

Insurance industry statistics demonstrate that cars with alarms or other antitheft devices are more likely to be stol...

hallerae March 31, 2018

C?

I still don't understand why the answer is C.

Replies
Create a free account to read and take part in forum discussions.

Already have an account? log in

Christy-Earls February 9, 2019

Same, can you please provide an explanation why C is the correct answer choice?

Ravi February 9, 2019

@hallerae and @Christy-Earls,

Happy to help.

The argument is saying that cars that have antitheft devices like
alarms are more likely to be stolen or broken into than cars that
don't have these anti-theft devices. The argument concludes that
anti-theft devices don't do anything to protect cars from thieves.

This argument is bad. What if the cars that have alarms are expensive
ones and ones that are parked in bad parts of town? In other words,
what if the fact that the cars are at high risk of theft is why they
have alarms? The argument fails to consider this.

The question stem asks, "The pattern of flawed reasoning in the
argument above is most similar to that in which one of the following?"

We're looking for a parallel flaw in the answer choice we select.

(C) says, "Studies reveal that people who are regular users of
libraries purchase more books per year than do people who do not use
libraries regularly. Hence using libraries regularly does not reduce
the number of books that library patrons purchase."

Similar to the stimulus, (C) presents a poor comparison. Just because
regular library users buy more books per year than people who don't
use libraries regularly doesn't mean that using libraries regularly
doesn't reduce the number of books that people buy. It could be that
frequent users of libraries would buy even more books if they didn't
go to libraries often. This answer choice fails to take this into
consideration, just as the stimulus failed to take into account the
alternative explanation for anti-theft devices in cars. As a result,
(C) matches the pattern of flawed reasoning we see in our stimulus
since both (C) and the stimulus fail to take into account potential
independent causes of the phenomena they're looking at.

Does this make sense? Let us know if you have any more questions!